La transformación de calibre del potencial vectorial multiplica la función de onda por fase

Considere un electrón en un campo electromagnético con potenciales escalares y vectoriales ϕ , A . Supongamos por simplicidad que A es independiente del tiempo. Supongamos también que conocemos la función de onda ψ de este electrón. Entonces ψ satisface

i ψ t = [ 1 2 metro ( pag ^ mi A C ) 2 + mi ϕ ] ψ = H ^ ψ

La pregunta se refiere a mostrar que si realiza una transformación de calibre de los potenciales:

A A = A + Λ
ϕ ϕ = ϕ 1 C Λ t = ϕ

para algún escalar Λ ( t , X ) , la función de onda se transforma como

ψ ψ = mi X pag ( i mi Λ C ) ψ
es decir, se multiplica por una fase. Es fácil demostrar que ψ satisface la ecuación de Schroedinger transformada:
i ψ t = H ^ ψ

Sin embargo, me gustaría saber si hay otras posibles soluciones a la ecuación anterior. Si es así, ¿Que son? o es ψ ¿la única solución?

Traté de encontrar otras soluciones suponiendo ψ = F ψ , dónde F es desconocido, y luego reemplazando esto en la nueva ecuación de Schroedinger. Esto da una nueva ecuación diferencial para F . Sin embargo, hasta ahora mis intentos de resolver esta ecuación diferencial han fallado.

Probablemente haya otra forma (¿quizás a través de integrales de ruta?) De mostrar esto de la que no estoy al tanto. ¿Podrías darme una pista, por favor?

Por favor aclare si su pregunta es esta: Si ψ y ψ satisfacer las ecuaciones de Schroedinger respectivamente
(01) S C h r ( ψ , A , ϕ ) = 0 (01') S C h r ( ψ , A , ϕ ) = 0
y
(02) ψ = mi i mi Λ ψ
entonces
(03a) A = A + Λ (03b) ϕ = ϕ Λ t
No exactamente. Mi pregunta es: si empezamos con (01), luego hacemos las transformaciones (03a) y (03b), cual es la función ψ que satisface (01'). Entonces mi pregunta cambia el orden de lo que estás proponiendo. En otras palabras, me gustaría demostrar directamente a partir de la ecuación diferencial (01') que ψ debe ser (02). Espero que esto sea más claro a partir de mis ediciones.
Esto es realmente fácil de ver en el caso relativista, ya que la ecuación es lineal en cantidad de movimiento. O en lagrangiano: esencialmente, lo mismo. Traté un poco de derivar esto en su caso no relativista, pero las matemáticas salen mal. Quiere exactamente el hamiltoniano especificado, ¿verdad?

Respuestas (2)

Esta respuesta está motivada por el efecto Aharonov-Bohm y prueba lo que pide el OP, pero en el caso especial

(01) × A = 0 = × A eso es B = 0

Para simplificar las expresiones hacemos:

  1. colocar

    (02) = 1 , C = 1 , mi = 1 , metro = 1 2

  2. use un punto para la derivada parcial con respecto a t

    (03) ψ ˙ ( X , t ) ψ ( X , t ) t

  3. omitir la dependencia ( X , t ) a menos que sea necesario.

Ahora, de acuerdo con OP, sabemos que si a la ecuación de Schroedinger de una partícula en campo electromagnético [ A ( X , t ) , ϕ ( X , t ) ]

(04) i ψ ˙ = [ ( i A ) 2 + ϕ ] ψ

reemplazamos la función de onda ψ ( X , t ) por

(05) ψ ( X , t ) = mi i Λ ( X , t ) ψ ( X , t ) eso es hacer la sustitución ψ mi i Λ ψ

entonces esta nueva función de onda obedece a la ecuación de Schroedinger de una partícula en un campo electromagnético [ A ( X , t ) , ϕ ( X , t ) ]

(06) i ψ ˙ = [ ( i A ) 2 + ϕ ] ψ

dónde

(07a) A = A + Λ , con Λ ( X , t ) R (07b) ϕ = ϕ Λ ˙

eso es en resumen

(08) ( i ψ ˙ = [ ( i A ) 2 + ϕ ] ψ ψ ( X , t ) = mi i Λ ( X , t ) ψ ( X , t ) ) ( i ψ ˙ = [ ( i A ) 2 + ϕ ] ψ A = A + Λ , ϕ = ϕ Λ ˙ )

Nota: La prueba de esta declaración se encuentra en libros de texto y en la web: http://www.physicspages.com/2013/02/01/electrodynamics-in-quantum-mechanics-gauge-transformations/

La pregunta, en su 2ª versión como en el comentario de RPF, es la inversa de (08) en el siguiente sentido:

(09) ( i ψ ˙ = [ ( i A ) 2 + ϕ ] ψ i ψ ˙ = [ ( i A ) 2 + ϕ ] ψ A = A + Λ , ϕ = ϕ Λ ˙ ) ??? ( ψ ( X , t ) = mi i METRO ( X , t ) ψ ( X , t ) METRO ( X , t ) R )

Ahora si ψ ( X , t ) obedece (04) bajo la condición (01) entonces

(10) ψ ( X , t ) = ψ 0 ( X , t ) Exp [ i Γ A ( X , t ) d X ]

dónde Γ ( X ) caracteriza una curva arbitraria en un espacio tridimensional que parte de cualquier punto constante X 0 y termina en el punto X , como en la figura, y ψ 0 ( X , t ) representa una solución de la ecuación de Schrödinger (04) con A = 0 pero por lo demás arbitrario ϕ ( X , t ) , que obedece a la ecuación de Schrödinger reducida

(11) i ψ ˙ 0 = [ ( i ) 2 + ϕ ] ψ 0

De la misma manera después de la transformación (07) y dado que la nueva función de onda obedece a (06) bajo la condición todavía válida (01) entonces

(12) ψ ( X , t ) = ψ 0 ( X , t ) Exp [ i Γ A ( X , t ) d X ]

dónde Γ ( X ) caracteriza una curva arbitraria en un espacio tridimensional que parte de cualquier punto constante X 0 y termina en el punto X , como en la figura, y ψ 0 ( X , t ) representa una solución de la ecuación de Schrödinger (06) con A = 0 pero por lo demás arbitrario ϕ ( X , t ) [ = ϕ ( X , t ) Λ ˙ ( X , t ) ] , que obedece a la ecuación de Schrödinger reducida

(13) i ψ ˙ 0 = [ ( i ) 2 + ϕ ] ψ 0

Vamos ahora a la transformación de calibre.

(14) ( i ψ ˙ 0 = [ ( i 0 ) 2 + ϕ ] ψ 0 ξ ( X , t ) = mi i Λ ( X , t ) ψ 0 ( X , t ) ) ( i ξ ˙ = [ ( i A ξ ) 2 + ϕ ξ ] ξ A ξ = 0 + Λ , ϕ ξ = ϕ Λ ˙ = ϕ )

esa es la funcion de onda ξ ( X , t ) obedece a la ecuación de Schrödinger

(15) i ξ ˙ = [ ( i Λ ) 2 + ϕ ] ξ

La condición (01) también se cumple para (15)

(dieciséis) × A ξ = × Λ = 0

entonces en analogía con los pares de ψ -ecuaciones (10)-(11) y ψ -ecuaciones (12)-(13)

(17) ξ ( X , t ) = ξ 0 ( X , t ) Exp [ i Γ ξ A ξ ( X , t ) d X ] = ξ 0 ( X , t ) Exp [ i Γ ξ Λ ( X , t ) d X ]

dónde Γ ξ ( X ) caracteriza una curva arbitraria en un espacio tridimensional que parte de cualquier punto constante X 0 ξ y termina en el punto X , como en la figura, y ξ 0 ( X , t ) representa una solución de la ecuación de Schrödinger (15) con A ξ = 0 pero por lo demás arbitrario ϕ ( X , t ) , que obedece a la ecuación de Schrödinger reducida

(18) i ξ ˙ 0 = [ ( i ) 2 + ϕ ] ξ 0

Pero (18) para ξ 0 ( X , t ) es idéntica a (13) para ψ 0 ( X , t ) para que podamos identificar las dos funciones y así

(19) ξ 0 ( X , t ) ψ 0 ( X , t )
Combinando (12), (19), (17) y la ecuación inferior entre paréntesis izquierdos en (14), es decir ξ = Exp [ i Λ ] ψ 0 , tenemos

(20) ψ ( X , t ) = mi i METRO ( X , t ) ψ ( X , t ) (21) METRO ( X , t ) = Λ ( X , t ) + Γ A ( X , t ) d X Γ A ( X , t ) d X Γ ξ Λ ( X , t ) d X

Si se selecciona el punto de inicio de cualquier curva, la integral de fase relativa es independiente de la trayectoria, ya que la función vectorial bajo la integral tiene un rotacional cero. El primer y último término de la derecha de (21) dan

(22) Λ ( X , t ) Γ ξ Λ ( X , t ) d X = Λ ( X , t ) [ Λ ( X , t ) Λ ( X 0 ξ , t ) ] = Λ ( X 0 ξ , t )

si elegimos X 0 X 0 entonces los términos 2 y 3 de la derecha de (21) dan

Γ A ( X , t ) d X Γ A ( X , t ) d X = Γ Λ ( X , t ) d X + Γ Γ A ( X , t ) d X 0 (23) = Λ ( X , t ) Λ ( X 0 , t )

Por las ecuaciones (22) y (23) la ecuación (21) da

(24) METRO ( X , t ) = Λ ( X , t ) Λ ( X 0 , t ) + Λ ( X 0 ξ , t )

Finalmente si elegimos X 0 ξ X 0 entonces

(25) METRO ( X , t ) = Λ ( X , t )
ingrese la descripción de la imagen aquí

Referencia: EJEMPLO 1.6 El efecto Aharonov-Bohm en "Mecánica cuántica - Capítulos especiales" de Walter Greiner, 1998 Edición en inglés.

¿No es esto una exageración?
@Frobenius: ¿Puede responder mi pregunta en: physics.stackexchange.com/questions/264916/…
OK. Entonces no contestes. Solo diga (a través de comentarios) dónde me equivoqué en mi pregunta.
@Frobenius: Ya terminé de desenterrar el tratado de Maxwell sobre la "Ley de la fuerza del amperaje". Ahora quiero saber algo que no está en detalle en el tratado de Maxwell; eso es sobre la Tercera Ley de Newton en forma débil (reacción de acción igual) en las cuatro ecuaciones de fuerza. Podría estar en algún lugar equivocado en mi pregunta. Por eso termino sacando la conclusión equivocada, es decir, habría una fuerza igual y opuesta en los cuatro casos. Necesito que el contestador señale dónde me equivoco.
@Frobenius: Planeo poner la derivación de la Ley de Fuerza de Ampere en un sitio web en un lenguaje fácil de entender. Busqué en Internet durante más de dos meses una derivación de la Ley de Fuerza de Ampere y solo pude encontrarla en el Tratado de Maxwell. Puede ser útil para aquellas personas que deseen conocer una derivación de la Ley de Fuerza de Ampere. Estas teorías olvidadas del electromagnetismo deben ser revividas.

Puedo mostrar esto al revés y luego explicar la motivación. La unicidad de la solución se deriva de la restricción de que la transformación no sea física.

Digamos, tomamos tu ecuación y transformamos ψ :

i t ψ = ( Π 2 2 metro + mi ϕ ) ψ

se convierte

i t ( mi i mi C Λ ψ ) = ( Π 2 2 metro + mi ϕ ) mi i mi C Λ ψ

Vamos a evaluar esto en etapas.

i t ( mi i mi C Λ ψ ) = i t ( mi i mi C Λ ) ψ + i mi i mi C Λ t ψ = mi i mi C Λ ( mi C t Λ ψ + i Λ t ψ )

Ahora, considere el efecto de Π ^ = pag ^ mi C A en el exponente.

pag ^ mi i mi C Λ = i mi i mi C Λ = mi i mi C Λ ( Λ + pag ^ )

(debido a la regla de diferenciación de Leibnitz)

Al jugar con esto durante algún tiempo, puede demostrar que

Π ^ 2 mi i mi C Λ = mi i mi C Λ ( Π ^ + mi C Λ ) 2

Aquí ya puedes ver cómo mi i mi C Λ cancela de ambos lados de la ecuación y cómo las adiciones se absorben en ϕ y A . Básicamente, conmutamos el exponente con operadores diferenciales para lograr esto.


Lo racional

La simetría de calibre se conocía a partir de la electrodinámica clásica, por lo que tuvo que incorporarse a la mecánica cuántica. Sin embargo, a priori no está claro cómo deberían entrar en la ecuación los potenciales electromagnéticos. La inspiración en la ecuación anterior proviene de la mecánica clásica, donde las ecuaciones de movimiento correctas se pueden lograr con la forma del hamiltoniano que vemos (p. ej., aquí ; la invariancia de calibre se usa implícitamente).

Pero si aplicamos la transformación de calibre directamente, obtenemos un lío en nuestra ecuación de Schrödinger. Pero esperamos que esta transformación no tenga consecuencias físicas, por lo que todos los observables deben permanecer iguales. El estado del sistema está definido por su función de onda, pero solo la amplitud es física, mientras que su fase no es observable.

Por lo tanto, queremos absorber los términos excesivos en la ecuación en el cambio de fase dependiente de las coordenadas de la función de onda que no nos da más opción que la transformación anterior.

Entonces, esta es la única transformación que no influye en la amplitud.